lsatforums
Thanks Received: 1
Vinny Gambini
Vinny Gambini
 
Posts: 1
Joined: April 03rd, 2014
 
This post thanked 1 time.
 
 

Q4 - The theory of military deterrence

by lsatforums Tue Feb 21, 2012 12:03 am

If you want to deter nations from attacking you, you must be believed to be so powerful that they would "have reason to believe" they couldn't withstand your retaliation. In other words, you have to look so tough that they're scared to attack you.

So what can we infer from this?

I see why "interest" threw you off on (D), because you're right; the stimulus doesn't discuss what's in the best interests of nations. HOWEVER, this answer choice takes care of that problem by saying "It is in the interests of a nation that seeks deterrence and has unsurpassed military power..."

So we're told everything we need to know about this hypothetical nation: it wants to deter its enemies, so we know what its interests are; and it actually has unsurpassed military power. We can infer that such a nation would want to let potential aggressors know how strong it is, because that would give them reason to believe they couldn't withstand retaliation; in other words, that would meet the necessary condition of military deterrence.

(A) is too strong; the aggressors don't need "certain knowledge," they just need "reason to believe."

(B) is the opposite of what's going on. The point is not that the strong nation won't attack, it's that the weaker one won't attack.

(C) is a mistaken negation or reversal, depending how you want to look at it. Just because the nation hasn't attacked (sufficient condition) doesn't mean it's because they're scared (necessary condition). Maybe they're not scared, but they just don't have any reason to attack the other country!

(E) is also too strong: the nation doesn't have to be the strongest, it just has to appear strong enough to make the other nation not want to attack.
User avatar
 
Mab6q
Thanks Received: 31
Atticus Finch
Atticus Finch
 
Posts: 290
Joined: June 30th, 2013
 
 
 

Re: Q4 - The theory of military deterrence

by Mab6q Mon Nov 17, 2014 9:21 pm

D is definitely the safer choice, but I want to push A a bit here. If it has said "believed to have retaliatory.." instead of "certain knowledge" would it be correct? I want to know if besides that issue the logic matches up.
"Just keep swimming"
 
christine.defenbaugh
Thanks Received: 585
Atticus Finch
Atticus Finch
 
Posts: 536
Joined: May 17th, 2013
 
 
 

Re: Q4 - The theory of military deterrence

by christine.defenbaugh Sun Nov 23, 2014 5:25 pm

Interesting question, Mab6q!

The direction of the logic matches up, but there are 2 distinct instances of strong language in (A) that would give me pause. You've identified one of them!

Changing "certain knowledge" to something about a mere belief would fix that problem, but even then, the answer choice would then have the would-be aggressor believing that they "would be destroyed". We don't actually need them to believe that - we only need them to believe that the nation is so powerful that they "have reason to think" they couldn't defend themselves. That's quite a bit softer of a requirement than believing that they would definitely be destroyed.

Essentially, (A) is just way too strong in a few ways - we need an aggressor to believe they are in some danger, we don't them to be certain they will be destroyed.

Does that help clear things up a bit?
 
mjacob0511
Thanks Received: 6
Forum Guests
 
Posts: 32
Joined: September 02nd, 2013
 
 
 

Re: Q4 - The theory of military deterrence

by mjacob0511 Fri Apr 03, 2015 9:56 am

The problem I had with (E) is that the passage is only discussing "aggressor nations".

Therefore, maintaining maximum deterrence doesn't REQUIRE that the nation has the greatest retaliatory force. It could be there is a stronger nation, but it is not a potential aggressor and maximum deterrence will still be maintained with the second or third to greatest retaliatory force.

The above explanation that they just have to make nations believe they have the greatest force and not actually possess it is definitely satisfactory, but this is how I chose (D) over (E) on my test...
 
ying_yingjj
Thanks Received: 1
Jackie Chiles
Jackie Chiles
 
Posts: 28
Joined: March 12th, 2014
 
 
 

Re: Q4 - The theory of military deterrence

by ying_yingjj Tue Jul 28, 2015 7:10 pm

The reason why I did not choose D is because:

It is in the interested of a nation that has unsurpassed military power to let potential aggressors against it become aware of its power.

"has unsurpassed military power" is not required. What is required is that "would have to be believed".

Although North Korea does not have the unsurpassed military power, but since they keep firing missiles so that they made the U.S. to believe that the U.S. may not be able to defend (or afford) against such retaliation.

So, actually having or not having unsurpassed military power is not required, so it is not in the interests of a nation. What in the interest of the nation is to make the potential aggressor to believe that they have the power to retaliate.

I am just not into LSAC questions, you can find those imprecision and yet is the most important score to be valued to get into law school. Nonsense.
 
burqin
Thanks Received: 0
Vinny Gambini
Vinny Gambini
 
Posts: 9
Joined: April 23rd, 2016
 
 
 

Re: Q4 - The theory of military deterrence

by burqin Sat Apr 23, 2016 10:22 am

ying_yingjj Wrote:The reason why I did not choose D is because:

It is in the interested of a nation that has unsurpassed military power to let potential aggressors against it become aware of its power.

"has unsurpassed military power" is not required. What is required is that "would have to be believed".

Although North Korea does not have the unsurpassed military power, but since they keep firing missiles so that they made the U.S. to believe that the U.S. may not be able to defend (or afford) against such retaliation.

So, actually having or not having unsurpassed military power is not required, so it is not in the interests of a nation. What in the interest of the nation is to make the potential aggressor to believe that they have the power to retaliate.

I am just not into LSAC questions, you can find those imprecision and yet is the most important score to be valued to get into law school. Nonsense.


It's not imprecision but a vicious trap. Normally "unsurpassed" is a glaring "too strong" or "wrong" signal. So the test taker could be readily misled. The standard answer D could have been "It is in the interested of a nation to let potential aggressors against it become aware of its power." The test maker add a descriptive clause "that has unsurpassed military power " for "a nation". The descriptive clause is actually not "required" but rather "optional". Whether or not the clause appears, the logic is not affected even a bit. Here "It is in the interested of a nation that has unsurpassed military power to let potential aggressors against it become aware of its power." is logically identical to "It is in the interested of a nation that has a miserable military power to let potential aggressors against it become aware of its power."
 
donghai819
Thanks Received: 7
Elle Woods
Elle Woods
 
Posts: 65
Joined: September 25th, 2015
 
 
 

Re: Q4 - The theory of military deterrence

by donghai819 Tue May 31, 2016 5:09 pm

The structure of choice D is pretty much like a conditional statement; it could be read as if:
If a nation is interested in seeking deterrence and has unsurpassed military power, it should let potential aggressors know its capacity.
Then it becomes much more plausible as a correct choice b/c of the conditionality.
 
WeiW222
Thanks Received: 0
Vinny Gambini
Vinny Gambini
 
Posts: 3
Joined: December 18th, 2017
 
 
 

Re: Q4 - The theory of military deterrence

by WeiW222 Thu Jan 04, 2018 12:01 pm

Can someone pls comment more on (C)?? It seems to me that it actually matches the conditional statement in stimulus (military deterrence -> believed to have retaliatory power. I don't see how it is a MR or MN. Thanks!
 
JasonZ200
Thanks Received: 0
Vinny Gambini
Vinny Gambini
 
Posts: 1
Joined: January 09th, 2018
 
 
 

Re: Q4 - The theory of military deterrence

by JasonZ200 Tue Jan 09, 2018 11:49 am

WeiW222 Wrote:Can someone pls comment more on (C)?? It seems to me that it actually matches the conditional statement in stimulus (military deterrence -> believed to have retaliatory power. I don't see how it is a MR or MN. Thanks!


(C) is a classic logic mistake. The proposition that "If A, then B" is not equal to "If B, then A." If a state has reason to believe it cannot defend the retaliatory attack, it will not conduct the attack - is not equal to - If s State choose not to attack, it must be that it believes that it cannot defend the retaliatory attack.
 
kh835
Thanks Received: 0
Vinny Gambini
Vinny Gambini
 
Posts: 1
Joined: December 20th, 2018
 
 
 

Re: Q4 - The theory of military deterrence

by kh835 Thu Dec 20, 2018 9:47 am

Just add one possible explanation to answer A. I think the phrase "only if" is an extreme restriction. It becomes problematic when it comes to the final part which says it would be destroyed in retaliation by the country it attacks. Since if the ally or any other state says hey you cannot attack my ally or this state because we have a deal about national security, then the country might still be deterred. In the case of North and South Korea, if the US or China say you just mustn't do this missile stuff or we will protect the SK and destroy your regime then it's possible that NK will be deterred.
 
HumphreyY750
Thanks Received: 1
Vinny Gambini
Vinny Gambini
 
Posts: 4
Joined: October 14th, 2019
 
This post thanked 1 time.
 
 

Re: Q4 - The theory of military deterrence

by HumphreyY750 Sun Nov 10, 2019 6:13 pm

I did not choose D in the first run but would agree with other people that D is the only choice one could choose.


D is a subset of the generalization of It is in the interested of a nation that seeks deterrence to let potential aggressors against it become aware of its power of retaliatory attacks.

It must be true logically and all the other choices are logically incorrect.

I don't think this could be reasonably argued against.